Können wir das Schwarze Loch (theoretisch) so stark drehen, dass es durch Zentrifugalkraft zerbrochen wird?


26

Ich kann mir nicht vorstellen, welche Kräfte in das Leben der Schwarzen Löcher verwickelt sind. Bitte helfen Sie mir herauszufinden, ob es möglich ist, ein Schwarzes Loch auf diese Art und Weise zu zerstören.


Ich bin mir nicht sicher, aber ich denke, um den Spin effizient zu steigern, musst du etwas darauf werfen, und wenn du das tust, wirst du auch die Masse erhöhen. in der starken Feldgrenze von GR verhindert der entsprechende Massenanstieg, dass die Zentrifugalkraft sie aufbricht. Bestenfalls handelt es sich um ein Kerr-Schwarzes Loch mit einem asymptotisch nahe an 1 liegenden "A". Bei Schwarzen Löchern gewinnt immer die Schwerkraft! (wenn du es aufdrehen willst, ohne Dinge zu werfen, musst du ziemlich weit weg sein und es wird nicht sehr effizient sein)
chris

Antworten:


23

Können wir das Schwarze Loch (theoretisch) so stark drehen, dass es durch die Zentrifugalkraft zerbrochen wird?

Für ein Kerr-Newman (rotierendes, geladenes, isoliertes) Schwarzes Loch der Masse , des Drehimpulses J und der Ladung Q ist die Oberfläche des Ereignishorizonts gegeben durch A = 8 M [ M 2 + ( M 2 - a 2 - Q 2 ) 1 / 2 - Q 2 / 2 ] , wobei a = J / M . Ein extremes Schwarzes Loch tritt auf, wenn M 2 = a istMJQ

A=8M[M2+(M2a2Q2)1/2Q2/2],
a=J/M . Wenn das Schwarze Loch noch mehr übersponnen oder überladen ist, ist es darüber hinaus eine "überextreme" Kerr-Newman-Raumzeit, die eigentlich gar kein Schwarzes Loch, sondern eine nackte Singularität wäre.M2=a2+Q2

Ich interpretiere Ihre Frage daher als die Frage, ob ein Schwarzes Loch bis an die äußerste Grenze und darüber hinaus gesponnen werden kann, um den Ereignishorizont zu zerstören. Es ist sehr wahrscheinlich, dass dies nicht möglich ist.

Wald hat 1974 bewiesen, dass es umso schwieriger ist, diesen Prozess fortzusetzen, je mehr man in ein Schwarzes Loch schleudert, um seinen Drehimpuls zu erhöhen würde es jenseits der äußersten Grenze nehmen. Es gibt andere Schemata, und obwohl mir kein allgemeiner Beweis für die klassische allgemeine Relativitätstheorie bekannt ist, ist das andauernde Scheitern solcher Schemata durch den Zusammenhang zwischen der Dynamik des Schwarzen Lochs und der Thermodynamik gut motiviert.

TH=κ/2π

κ=M2a2Q22M(M+M2a2Q2)Q2

2

Ich habe nicht die ganze Mathematik im Kopf, aber nach meinem konzeptuellen Verständnis ist es nicht möglich.

Schwarze Löcher haben eine ausreichend große Anziehungskraft, so dass auch Licht nicht weit über die "Oberfläche" hinaus entweichen kann (dh, wenn das Schwarze Loch eine ausreichend geringe Masse hat, dass es noch eine Oberfläche hat und nicht zu einer Singularität zusammengebrochen ist). Das würde bedeuten, dass es sich schnell genug drehen müsste, damit sich die Oberfläche deutlich schneller als die Lichtgeschwindigkeit bewegt, um genügend linearen Impuls (oft umgangssprachlich "Zentrifugalkraft" in einem kreisförmigen Bezugsrahmen genannt) zu haben, um zu entweichen Die Relativitätstheorie ist nicht möglich.

Hawking-Strahlung ist nur möglich, weil sich die elektromagnetische Strahlung nahezu orthogonal zur "Oberfläche" des Schwarzen Lochs bewegt und das Licht nur durch die Schwerkraft "gebogen" werden kann, es kann nicht zum Stillstand gebracht werden.


2
Es gibt verschiedene konzeptionelle Probleme mit dieser Antwort. Es deutet darauf hin, dass die "Oberfläche" etwas anderes als der Ereignishorizont ist - was ist es dann? Es legt nahe, dass Hawking-Strahlung auf masselose Strahlung wie elektromagnetische Strahlung beschränkt ist - nicht wahr. Es deutet darauf hin, dass Schwarze Löcher das Licht nicht aufhalten können - der Horizont ist eine lichtähnliche Oberfläche, die dies tut.
Stan Liou


0

Soweit wir wissen, gibt es nichts, was ein Schwarzes Loch aufhalten könnte. Damit dieser Begriff Sinn ergibt, müssen Sie sich zunächst ansehen, was derzeit über Schwarze Löcher bekannt ist . Wenn Sie sich erst einmal ein Bild davon gemacht haben, werden Sie feststellen, dass wir aufgrund unseres derzeitigen Verständnisses des Kosmos nichts gegen Schwarze Löcher unternehmen können.

Es ist wahr, dass die Hawking-Strahlung ein Schwarzes Loch beeinflussen kann, aber das gilt nur für sehr, sehr kleine Schwarze Löcher.

Übrigens gibt es in der Physik keine Zentrifugalkraft - dies ist tatsächlich ein Missverständnis, das viele Menschen haben. Es gibt jedoch Zentripetalkraft .

Bildbeschreibung hier eingeben


2
Obligatorischer Rebuttal-via-xkcd-Link: xkcd.com/123
Ilmari Karonen

3
@IlmariKaronen Danke für die Abwahl, aber du merkst doch, dass der Cartoon nichts richtig macht? Es gibt einen Unterschied zwischen der Zentripetalkraft und der Zentrifugalkraft.
FunctionR

2
Um fair zu sein, weist der Cartoon darauf hin, dass die Zentrifugalkraft eine Trägheitskraft ist, was wahr ist. Es ist wohl sinnvoll, Trägheitskräfte als "nicht real" zu interpretieren, aber ich bin mir nicht sicher, welchen Einfluss dies auf diese Frage hat, da die Gravitationskraft auch eine Trägheitskraft ist.
Stan Liou

Diese "Antwort" ist ziemlich vage, macht fragwürdige Aussagen und hat keinen wirklichen Einfluss auf die ursprüngliche Abfrage.
Florin Andrei

0

Interessant. Dieser Prozess könnte sich zunächst auf die Bildung von Schwarzen Löchern auswirken. Stellen Sie sich einen rotierenden Stern vor, der stirbt und aufgrund der Gravitationskräfte zu schrumpfen beginnt. Während es schrumpft, wird seine gesamte Masse in einem kleineren Radius immer mehr verdichtet. Dies hat zwei Konsequenzen: 1) Die Gravitationskraft, die verschiedene Körperteile anzieht, wächst mit dem Kehrwert des quadratischen Radius und 2) Die Drehzahl steigt aufgrund der Erhaltung des Drehimpulses und der Expansionskraft aufgrund der Drehung. wächst mit dem Kehrwert des gewürfelten Radius auf. Dies bedeutet, dass die expandierende Kraft schneller wachsen wird als die kontrahierende, und zumindest nach Newtons Ansicht wird die expandierende Kraft gewinnen. Unter diesem Gesichtspunkt sieht es so aus, als würde ein rotierender Stern niemals ein Schwarzes Loch bilden ...


Dieser Artikel könnte Ihnen gefallen. slate.com/blogs/quora/2014/02/10/…
userLTK

-1

Versuchen wir Folgendes:

Gleiche die Kräfte:

Fc=mv2RFG=GMmR2mv2/R=GMmR2v2=GMR

Rs=2GM/c2

v2=GM2GM/c2v=c2/2v=c2v=0,707c

Ich weiß nicht, ob ich alle Überlegungen angestellt habe, aber soweit ich weiß, dreht sich das Schwarze Loch schneller als 0,707ckann sich der Ereignishorizont nicht auf dem aktuellen Radius halten.

Wenn sich der Radius jedoch erweitert, verlangsamt sich die Rotation durch die Beibehaltung des Drehimpulses. Ich glaube also nicht, dass er zerreißt. Vielleicht wird er zu einem "grauen Loch"?

Bitte verzeihen Sie mir, wenn ein grundlegender Fehler gemacht wurde, ich bin neu in all dem ...: P


3
Das sieht nach Newton aus. Haben Sie die Krümmung der Raumzeit in der Nähe eines Schwarzen Lochs betrachtet? Siehe auch "Kerr-Lösung" für Schwarze Löcher.
Gerald

Ich mag den Begriff "Graues Loch".
userLTK

Dies ist eine alte Antwort, aber ich dachte, ich würde dies hier fallen lassen, weil es die Umlaufgeschwindigkeit von Schwarzen Löchern anspricht. Sie müssten die theoretische Kante des Schwarzen Lochs am Ereignishorizont bei C drehen, um zu entkommen, was offensichtlich unmöglich ist. physics.stackexchange.com/questions/207816/…
userLTK
Durch die Nutzung unserer Website bestätigen Sie, dass Sie unsere Cookie-Richtlinie und Datenschutzrichtlinie gelesen und verstanden haben.
Licensed under cc by-sa 3.0 with attribution required.